0 Daumen
615 Aufrufe

Aufgabe:

Mathematik: Lagrange Methode


Problem/Ansatz:

Gegeben sei die Matrix A= (1 0,5  
                                          0,5 1)


und die Menge G={⃗v∈R2 |⃗vTA⃗v=1}.
Bestimmen Sie die Punkte aus G, die den minimalen und maximalen Abstand zum Koordina- tenursprung haben! Nutzen Sie die Lagrange-Methode. Ich verstehe nicht ganz was man machen muss nachdem man die Haupt und Nebenbedingung gefunden?

Avatar von

'Menge G={⃗v∈R2 |⃗vTA⃗v=1}' ist nicht lesbar.

Mein Fehler : hier nochmal G :


blob.png

1 Antwort

+2 Daumen
 
Beste Antwort

Aloha :)

Der Abstand eines Punktes \(\vec v=\binom{x}{y}\) vom Koordinatenursprung ist \(d(x;y)=\sqrt{x^2+y^2}\). Dieser soll unter der Nebenbedingung, dass der Punkt \(\vec v\) zur Menge \(G\) gehört optimiert werden. Anstatt der Wurzel können wir auch das Quadrat des Abstandes optimieren (weniger Rechenaufwand):$$f(x;y)=x^2+y^2\to\text{Extremum!}$$\(\vec v\) gehört genau dann zu \(G\), wenn gilt:$$1=\vec v^T A\vec v=\begin{pmatrix}x & y\end{pmatrix}\begin{pmatrix}1 & \frac{1}{2}\\\frac{1}{2} & 1\end{pmatrix}\binom{x}{y}=\begin{pmatrix}x & y\end{pmatrix}\begin{pmatrix}x+\frac{y}{2}\\\frac{x}{2}+y\end{pmatrix}=x\left(x+\frac{y}{2}\right)+y\left(\frac{x}{2}+y\right)$$$$1=x^2+xy+y^2$$Die Nebenbedingung zur Optimierung von \(f(x;y)\) lautet daher:$$g(x;y)=x^2+xy+y^2\stackrel!=1$$

Nach Langrange muss der Gradient der zu optimierenden Funktion proportional zum Gradient der Nebenbedingung sein. Der Proportionalitätsfaktor ist der Lagrange Multiplikator:$$\operatorname{grad}f(x;y)=\lambda\cdot\operatorname{grad}g(x;y)\implies\binom{2x}{2y}=\lambda\binom{2x+y}{x+2y}\implies\frac{2x}{2y}=\frac{\lambda(2x+y)}{\lambda(x+2y)}$$$$\implies2x(x+2y)=2y(2x+y)\implies 2x^2+4xy=4xy+2y^2\implies \underline{\underline{x^2=y^2}}$$

Dieses Ergebnis \(x^2=y^2\) setzen wir in die Nebenbedingung mittels einer Fallunterscheidung ein:

1. Fall: \(x=-y\):$$1=x^2+xy+y^2=x^2-x^2+x^2=x^2\implies x=\pm1\implies y=\mp1$$Wir erhalten zwei Kandidaten für Extrema:$$\underline{\underline{P_1(-1\big|1)\quad;\quad P_2(1\big|-1)}}$$

2. Fall: \(x=y\):$$1=x^2+xy+y^2=x^2+x^2+x^2=3x^2\implies x=\pm\frac{1}{\sqrt3}\implies y=\pm\frac{1}{\sqrt3}$$Wir erhalten zwei weitere Kandidaten für Extrema:$$\underline{\underline{P_3\left(-\frac{1}{\sqrt3}\bigg|-\frac{1}{\sqrt3}\right)\quad;\quad P_4\left(\frac{1}{\sqrt3}\bigg|\frac{1}{\sqrt3}\right)}}$$

Durch Einsetzen dieser Kandidaten in \(d(x;y)\) findet man für \(P_1\) und \(P_2\) mit \(\sqrt2\) die Maximalwerte und für \(P_3\) und \(P_4\) mit \(\sqrt{2/3}\) die Minimalwerte.

Avatar von 148 k 🚀

Sorry, dass ich mich so reingretsche, aber ich wollte fragen, wie man auf

2y/2x = λ(x+2y)/λ(2x+y) kommt?

Wir haben für jede Koordinate eine Gleichung:$$2x=\lambda(2x+y)\quad;\quad 2y=\lambda(x+2y)$$Wenn man die erste durch die zweite Gleichung dividiert, fällt \(\lambda\) raus.

Sorry, dass ich mich so reingretsche, aber ich wollte fragen, wie man auf

2y/2x = λ(x+2y)/λ(2x+y) kommt?


Vielen Dank für die schnelle Antwort!

Ein anderes Problem?

Stell deine Frage

Willkommen bei der Mathelounge! Stell deine Frage einfach und kostenlos

x
Made by a lovely community